Oxford Physics Aptitude Test: answers, discussion and advice

OCTOBER 2023

Well done class of 2023! Sorry for not being around much this Autumn; I am handing in my PhD thesis in about 10 days so I have been extremely busy. I hope it went well 🙂

If you feel you’ve got value from this site, please consider making a donation (or ask your parents). The solutions are and always will be free, but anything you’d like to give is appreciated. Due to the way the donations work I’m not able to reply to your kind messages but rest assured every donation comes with my heartfelt thanks.

This site is designed to help you succeed in the Oxford PAT, Oxford’s admission exam for the physical sciences.  We’ve got free scans of completed solutions to the question papers, visible one page at a time, with comments and links to background material where relevant. Accessing the content and asking questions is and always will be free. The blog format means you can post your own comments, get clarification of things you don’t understand, suggest improvements or corrections to our answers, and explain your thinking to others. You are encouraged to check other students’ questions to cut down on duplicates. Please remember to check the syllabus on the Oxford website. All questions are answered by me personally, a physics PhD student at Oxford.

Hi all!

I’m the moderator for Oxford PAT – I’m a current physics PhD student at Oxford, and did my undergrad/masters here too, so I’m familiar with the Oxford system. My Dad started this site because when I was doing the PAT in 2013 there were no free solutions available, and, even worse, lots of the paid ones were totally incorrect! Hopefully this site can be of some help to all of you.

Best,

M

2009 | 2010 | 2011 | 2012 | 2013 | 2014 | 2015 | 2016 | 2017 | 2018 

2019 | 2020 | 2021 | 2022

Sample papers   |   Changes to the syllabus

Here are some categories of question in case you want to revise something specific; these are not exhaustive, I haven’t got round to getting these updated yet:

Electricity | Mechanics | Problem Solving | Waves | Natural World

Geometry | Algebra | Calculus | Logarithms | Series | Graphs | Probability

Hard questions | Easy questions | Weird questions

Mail: oxfordpat.wordpress.com@gmail.com

545 thoughts on “Oxford Physics Aptitude Test: answers, discussion and advice

    1. From my understanding of the syllabus, we don’t need to know anything about magnetic fields (although weirdly, 1 or 2 multiple choice questions have come up since 2018). I’d just carry through your GCSE knowledge on magnetism and electromagnetism just in case. The only thing we need to know that is on the syllabus is the transformer equations.

      I’m not too sure on alternating current, but again, I don’t think I’ve come across a question on it, and we aren’t taught it at A-level, so I’d assume we aren’t really expected to work with it, unless it’s a “problem-solving” question. I’d just keep in mind the direction of the current switches periodically and other basic knowledge about it and you should be OK.

        1. Yeah I saw the tesla question and then Q9 on 2019 also had magnetism in it.

          Yes I’m applying this year. The online calculator is really annoying. What about you?

            1. The older papers seem relatively easy with the exception of the end of 2012. I’ve never really struggled with the mathematics side of any paper, it’s more about being careful about not making careless mistakes. My biggest points for improvement are being faster on the MCQ section, and just really tough Mechanics problems + waves.

                  1. I answered every question but missed a couple solutions for the apples bananas question and maybe made a few careless mistaked on the others. Hopefully 80+. What about you?

                    1. I’m applying for engineering so my cutoff should be lower than physics, but despite what people are saying on the 6thform reddit, I highly doubt it will be a lot over 70. It was a relatively easy paper content-wise, but was quite long still. I’d guess 72 or smt

    1. Hi Patrick!
      I’m actually going to be extremely busy finishing my PhD thesis until July, so the probability I finish this before late summer is essentially zero, sorry about that.

  1. hi, just wanted to say thanks so much for this website. I got an offer for physics in January from oxford and I don’t think I would have been able to without this site.
    thanks 🙂

  2. Ended with a 64, reject. Still got interviewed despite this score leaving a bit to be desired. This was a fun run though, hoping to go to Imperial now and take down Oxford in postgrad. Thank you for the website and see you in 2026 🙂

    1. That’s a shame but you will have an amazing time at Imperial, it really is an outstanding place. A good friend of mine went there and it was really every bit as good.

  3. Hi, I recently got an offer for physics and I can’t thank you enough for the solutions you put out. This site was a genuine lifesaver and I couldn’t have done without it.

    1. Ah rats, I hope you’re not too upset, that’s particularly unfortunate. It sounds like you worked really hard for the PAT and did very well, so if you forgive the optimism this will probably be totally fine long run. It can be down to all sorts of things why an interview didn’t go well, from not being familiar with the style to simple bad luck. However, you can pretty much only do well in the PAT by being genuinely quite good at physics. Not going to Oxford isn’t a big deal, being good at physics is. This means that wherever you go for univeristy will be lucky to have you and there’s every reason to think that if you continue working hard you will do extremely well. All the best to you – thank you for using my website!

      -M

  4. How would you guys rank the difficulty compared to previous versions? Maybe it was just the stress, but this year’s exam felt really hard.

  5. What did everyone get for that question with the 3 equations for x, y, and z… I got a general formula for x, but I’m not sure…

    1. I think I got x=0, x=2 and x=-4/3 after some rearrangement but I don’t really remember what I put. How well do you think you did?

      1. I initially walked out thinking I had done well, but then properly thinking about it I realised there were a few questions that I had got wrong, I’m hoping for 70 but I can’t remember all the questions and I’m not too confident in that…

        1. Initially I didn’t even think I got 50 but then thinking about it properly and I have a good chance of 70, pretty much guaranteed 60. Left the last one about radioactivity pretty much blank though

          1. I got the right exact answer for it, in terms of lns, but then my last line of working I forgot to times by the factor of 20…. and so my exact answer is right, but my approximate answer with decimals is wrong… sigh
            Which college have you applied to?
            Also, do you remember the questions and the associated marks, because quite a few seem to have slipped my mind… I think there were 11 long mark ones, the ones I remember are:
            Integration (messed up last part)
            Circles/spheres in a cube (got the sphere one wrong)
            Electron orbiting (do you remember the factor you had to times by for the last part?)
            Rolling balls on a frictionless surface (I said it changes in part b, not so sure anymore)
            Product of dice (happy with this)
            x, y, z simultaneous equations (stumped me)
            Radioactivity last question – got this apart from last silly mistake
            r(t) question with mass projectile being launched off a cliff (do you remember the total marks for this?)
            PQR question – think I got this all
            Jupiter satellite question – think I got this
            There seems to be 1 missing… also cant remember the marks for most of these lol

              1. I think I got something like 4P-12QR>=0 for the first part. Something similar for the second with a bit of derivatives.

      2. surely the answers for x would have to be in terms of z? 3 unknowns, 3 equations sure, but not all 3 of them had all 3 variables in, so there must be a y or z in the answers for x? I’m really not sure though I’m just tryna see If imma get in lmao

        1. guys I love the interaction but you are nuking my email inbox from orbit – there’s normally a discussion on thestudentroom I think, you might consider finding that! Hope it went well 🙂

        2. If you have 3 equations and 3 variables it is always possible to find the solutions, doesn’t matter if they don’t all have all of the variables or what order the equations are (as in squared, cubed, etc…) so you should have ended up with just numbers

      1. I mean the one with the two sphere masses on a frictionless plane colliding with each other. I knew how to do it my equating KE and momentum, but just didn’t have time 😦 The dice one I felt was pretty straightforward just by drawing out a diagram of 1-6 and their products.

        1. Did you also get 12 as the most likely outcome? The question was 2 marks but I can’t figure out what else the answer could be. I looked it up and 12 seems to be the only one. Also do you remember what you got for the divisible by 4 one?

          1. I got 6 as well; 6 x 1, 1x 6, 2 x 3, 3 x 2
            I really hate hindsight though… just thinking of the questions where I know what to do now but my mind skipped over it in the exam…

            1. Yep 6 is also right I never saw it argh. I was really wondering how it was 2 marks but didn’t have time to sit there and look through it properly. Hindsight hurts

    1. I somehow managed to end up with an imaginary speed for that, so just gave up. I should have picked up some marks though cos I did all that conservation business. For the charged part I just said it remains unchanged. Overall I found it okay, the radioactivity and electron ones kicked my behind though

      1. Gonna be honest I didn’t even read the second bit. just saw ‘charge’ and moved on. The last two bits for the electron orbit thing really stumped me as well. Couldn’t figure out how to get that equation in a form independent of v and r or whatever it was…

        1. Yeah the ball off a hill one hit hard. I tried going through it but kind of failed. I knew for a fact the maximum was at 45° and that distance travelled increases up to there. How many marks do you think you got? I thunk I pulled about 60

            1. I don’t remember what I got but I remember what I did. For the first part I set it equal to 0, then multiplied everything by x³ and did the discriminant. For the 2nd part I did roughly the same apart from doing the derivative too. For the last part I got that it’s a maximum. Is that what you did?

                1. For the first one I got Q²-4PR>=0 which is the same as you but with the 4PR moved over. It’s also =0 because the question said at least 1 real root. For the 2nd part I think I got 4Q²-12PR>=0, again same as you but unsimplified. I think we are right (hopefully they don’t dock marks for not simplifying)

                  1. Omg thank goodness hahahaha, yep hopefully your right!
                    What other questions did you think were hard btw? I think the projectile motion one I messed up ngl, aswell as the integral one

  6. Hi all!!! First of all, congrats for completing the PAT 🥳 as someone who went through the whole application process through to interviews last year, and not getting an offer at the end of it, I just wanted to say, best of luck! If you get in, well done! And if not, it is by no means a fail, I am so happy at my second choice, I’m sure I wouldn’t have enjoyed Oxford as much even. A rejection is merely a re-direction as you’ll hear many say! Simply getting this far is amazing!

    May each of us have a lovely time wherever we go, and go on to have a happy and fulfilling life!

    All the best,

    Andreea 🙂

    And many thanks again to oxfordpat for providing such immense help and support here! 👏

  7. I just finished the 2022 PAT, and I just want to thank you for the resources you upload here. I only found this page a week back, but man its been a invaluable resource. Wish I found it earlier but nonetheless I’m happy with how my paper went. Thank you so much, you’re doing a fantastic thing IMO and its so indispensable. Thank you!

  8. Hi, I am an Australian student doing the PAT and i cant find any comparison between our standard calculators that i use for all of school and the UK ones. From what I can tell ours are much the same function but they can be programmed to store data, would that disqualify them? You can easily delete all the stored data which is what we do before exams but i dont know if that will be an issue here

  9. Hi! I’m taking the PAT tomorrow and while I feel really stressed about it and feel in no way prepared, your website has been of tremendous help while revising and practising past papers! Thank you so much!

    1. Hi!

      You’re very welcome. Deep breath – the absolute worst that happens is it doesn’t go amazing, and honestly it’s just not that important in the grand scheme imo. My mum would often say ‘well, noone died’ if something not amazing happened and honestly it’s not a bad message to keep in mind. Stay calm and try your best 🙂

  10. I will be sitting the Pat which is in 2 days and i’m very stressed. Do you have any advice on what to do and also any tips for improving my physics as that’s the side where i lose most marks.

    Your website also has been very useful and i’ve very grateful.

    1. Hi Abdimaalik,

      I know this is ‘simple to say, hard to do’, but I would encourage you to take a deep breath and try to relax; you’ve been revising so hopefully you’ll be able to perform at your best. It’s a bit of a cliche but if you go into the exam and do your best, then that’s something you should always be proud of, however it turns out.

      Some specific advice for the physics bit: firstly, remember that physics is hard! I always say that the hard bit is translating a physics problem into a maths problem, then once you’ve done that it’s plug n chug.

      Secondly, some of the best advice is to check your units whenever you have a spare minute. If you reach the end of, say, a mechanics problem that asks you for the energy, but your answer has the units of distance then you’ve definitely made an error somewhere, and it can really help you to find problems.

      I asked my friends in the office if they had any ideas for advice; a good one was to bring a highlighter to use for the physics questions. He said he always found that helped him not get tangled up in the words and concentrate on extracting the maths.

      You’re very welcome, I’m glad it’s managed to help you. Good luck!

      1. Hi,
        Just wanted to say thanks for all the resources you have made on this website. I found the discussions under each question very useful!!
        Thank you very much 🙂

  11. Hi Oxford PAT,

    Any final advice before the exam? I’m roughly confident with the content and have practised enough to have a crack at the exam tomorrow. I feel like I should go over notes and questions a bit more, but is there anything you can add that would be useful and would really help (now and during the exam)?

    I think this would be helpful for all candidates taking the exam tomorrow.

    Thanks

    1. Honestly? Take the evening off. Breathe deep and have a bath, or spend time with your cat, read a book, or just do nothing. All the hard work is done (or should be) and you should let the information and skills seep in, which can only happen if you’re relaxed. Maybe have one last look over stuff in the morning over breakfast.

      1. Sounds great, I would also just like to thank you for everything. All the resources you’ve shared have been a huge help in my revision!

        1. Hi mate – sorry I’ve been on holiday for the past few weeks and didn’t update the page, I got home yesterday. If all goes according to plan I’ll do the paper tonight

  12. Hi there!

    Is it necessary to study resultant forces & angles for the exam? It’s a first-year Physics topic but a second-year Mathematics topic.

    Regards.

  13. Hi Oxford PAT. Thank you for this amazing website!
    I’m just wondering for the MCQs, if we just circle the correct answer without the need to show working out, do we still get 2 marks? Or do we have to write some working out in order to get the second mark? Thank you!

    1. Circling them should be fine but you should probably show some working. If the answer is obvious then it’s fine, but make an effort not to look like you just guessed right!

  14. Hi!

    Thanks for the brilliantly organised site.

    Quick question; Would we need to memorise the constants in the equations? E.g. Gravitational constant

    Thank you for your time.

  15. Hi, do you know if we need to know Y13 integration (integration by parts, by substitution etc.) and parametric equations for this year’s PAT. As it’s not explicitly on the syllabus but has come up in past papers (2017 I think)?

    1. As ever, the answer is ‘probably not’. They’re much better these days at not accidentally giving you things off syllabus. They aren’t massively hard so if you’re really worried about it you might as well go learn it.

  16. Hi, would you be able to update the link to waves questions? Every time I click on it, I just get redirected to problem-solving questions. Thank you!

  17. Hi there,

    Do you know if we need to know about differentiation from first principles? Thanks in advance!

  18. Hi!
    Do you know where can I find solutions for the 2009 specimen paper? I checked few sites, including physics and maths tutor, but it looks like it’s deleted there…
    Thanks!!!

  19. Hi,
    I am advising a Year 13 student who will be doing the PAT this year and was wondering if you could answer this please?
    If he takes the test this year and does not do well, will that prejudice his chance of applying again next year? He is undecided between Physics and Engineering, but as they both have the same test (I think?), does he have to specify which course on his application for the PAT?
    Finally, would he have the opportunity to give any extenuating circumstances with his test, or would that only go in his UCAS application?
    Thanks in advance if you are able to help us with any of these!
    Noeleen Ryan-Smith

  20. Hi,

    I’ve been using your solutions and they have been very helpful so thank you for setting up this page. Could I ask how you revised for the pat besides using the papers?

    1. Hi,
      I actually pretty much just used the past papers. I did do a bit more work on circular motion as my mechanics course hadn’t covered it to what I thought was a satisfactory level, so I swotted up on 1st year uni textbooks but that turned out to be unnecessary.

    1. You’re right, haven’t had that one before.

      I’m a mild fountain pen nerd – I think all of the recent (past few years) solutions were done with a Lamy Safari, medium nib. Day to day I use either that one or an extremely nice Pelikan Souverain that I found in a drawer in department. I handed it in but after a few months of nobody claiming it they said might as well keep it. The earlier solutions before I took over were from Dad, he just uses a biro.

      1. Hey,
        I am from India i have been trying to apply to OXFORD since 2 months but consultant isn’t responding and I cant find other to apply to PAT so, can please you please help for applying to OXFORD or give any advise on how to proceed from here.
        A single consultant responded and asked to pay 80,000 rupee (792Pound sterling) only for applying, I cant pay 792Pounds only for applying to it so can you please show me a way to apply to PAT exam I really really really want study material science at OXFORD.

  21. Hi, I’ve just stumbled across your site and think it’s absolutely amazing.
    My question is, how is a question marked? As in, do you get a mark for each stage of your working out? Or, would you get full marks if you just jotted down the answer and minimal working out? Etc,. I’ve tried looking for information released by Oxford and haven’t really found anything. Any information would be greatly appreciated.
    Thanks alot.

    1. Hi Hamza,
      Thank you! You’re welcome

      So first of all I don’t mark the PAT so I can’t say with any true accuracy. But, the point of the PAT is to assess your physics and maths thinking. If you somehow managed to write down an answer with no working, great, but how do I know you didn’t just guess it? When I’m marking undergraduate exams in their revision time the mark schemes have marks breaking up each question into sensible statements, not just ‘all or nothing’. Hope this helps.

      1. Thanks for the reply. That’s helpful. So if i were to only write some working out but was unable to attain the answer, would I still recieve any marks?

        1. Almost certainly yes! A small story told to me by one of my old tutors: one of his students was running out of time in finals and realised he’d screwed up a question, but knew what he should have done. He had enough time to leave a note saying ‘I’ve messed this up and run out of time!’, and described the exact method, answer, and reasoning, and ended up being awarded most of the marks even without having ‘properly’ answered the question. Nobody cares whether you know the answer, they care about you being able to work out the answer!

  22. When would the solutions be out for this years PAT? Like MAT has their solution out on Oxford Math YouTube channel already.

    1. When I get around to it. The Oxford MAT one is official, from the department itself; even though I’m a member of department this blog is not affiliated officially with Oxford so I don’t even have access to the paper yet. I’d say don’t hang around for it though, my schedule is to make sure they’re available for when people start revising for the next PAT.

      1. Ahh I see. Thanks a lot for the site btw. And by any chance do you know what would be a relatively safe mark to be shortlisted for an interview? I’m applying for engineering so is the PAT score required going to be lower than the one for Physics? What’s the weightage between the UCAS score and PAT score when being considered for an engineering degree. I tried searching up the reports for Engineering but there isn’t rly much out as most of them are for Physics. Thought you might know more since you’re part of the department. Thanks

        1. I’m afraid engineering is a different department! Have you tried trawling through their website? There might be something on admissions? Can’t possibly say what a ‘safe’ mark is as it definitely changes from year to year, but it seems to be that around 60-70 is the standard.

  23. Well that was a lot of questions :)) What do you recommend doing after finishing the PAT? Should we start preparing for the interview so we don’t lose what we have learned in Physics?

      1. This year they are in December, I am International Applicant from Germany. Have you also got Interview Advice/Resources?

        Thanks alot Oxford PAT! My PAT went well.

        1. I got an email from my college that made it clear that it was not an invitation to interview, haha! They gave me a link to a website specifying the technology needed and frequently asked questions related to the online interview.

          Did you already get the results and where do you see them?

  24. Hi! I just finished the PAT test 2020. I wonder if you there is any place where I can find or discuss the answers

  25. Just to let you know, when you click on the Waves link to see the Waves questions, it redirects to Problem Solving. Also is there no category for optics or does that just fall into waves?

    1. Huh, thanks Kainaat. Yeah that part of the site is ‘currently’ undergoing an overhaul, it’s never really been what I wanted it to be. I’ll get around to that eventually, thanks for pointing it out.

  26. Hi, can you say me where I can read about the most essential knowledge for PAT about the moving of planet and eclipses?
    And thank you a lot for this site!!!

  27. If we use an incorrect answer from a previous part to solve a subsequent part, will we get credit if we used the answer correctly?

    1. I’d imagine so, yes. It would be overly harsh to punish a slip up carried forward. However, it might not be possible to get full marks on the rest of the question if the incorrect starting point makes you conclude something radically different!

  28. Hello! Will we have to simplify our fractions for our answers? For example, say the answer is (2pix(sqrt(3)))/3. Would we be fine if we said something like pi(sqrt(4x^2))/sqrt(3)? Do we have to simplify the fractions?

    1. I wouldn’t say that it’s an absolute necessity but in my opinion, unsimplified fractions look sloppy and are hard to read. Your aim is to make it easy for the examiner to give you marks.

    1. I don’t think so. If it’s not on the a-level syllabus then I’d be fairly confident in it not coming up. That said, it’s totally fair game for them to give you the information and ask you to solve a question about it.

  29. There are some questions involving Kepler’s laws but there is no mention of it on the syllabus and I haven’t really learned them. Is there a chance they would come up or has the spec changed so that I don’t have to worry?

    1. Hi Kainaat,
      It’s hard to know exactly what you mean. Kepler’s laws used to be on syllabus, yes, but apparently aren’t anymore (You can check the syllabus online!). That said, I’ve always warned students that the syllabus isn’t necessarily 100% accurate. I’d say don’t worry. Although, Kepler’s laws are interesting! If you have a spare hour, you might as well learn them.

  30. Hey, thanks for this website! I was wondering how the examiners evaluate the test. Would they give you less than the total score of a question if you just write down the answer without the approach?
    And do we need to know integration by substitution and partial integration?
    In which way has the syllabus changed concerning sin/cos and tan and radiants? Bc the older Pats (around 2010) have a lot of higher sin cos and tan in them, and radiants.
    Thank you!

    1. I’m afraid I can’t answer with total confidence as I don’t mark it.
      1) My gut feeling is yes, absolutely. The process is more important than the answer. The point of the PAT is, much like the interview, to determine how you think, and whether how you think is compatible with what Oxford wants.
      2) Integration by substitution has been useful in the past. I don’t know if it’s strictly necessary, but I’m assuming everyone doing the PAT knows it? Integration by parts has never been necessary to the best of my knowledge.
      3) I have no idea. They’re probably just set by different people. None of the questions require any special knowledge, it just changes what the relative focus is on so I really wouldn’t worry about it.

  31. Is conic section part of PAT? Has any question ever come from ellipse, parabola or hyperbola in the context of Co-ordinate Geometry?

    1. I believe not, at least nothing that required any special knowledge beyond the familiarity you’ll have picked up in maths classes. As ever, the syllabus is online so have a check of that.

  32. Hi,
    Are we expected to know how to integrate and differentiate trig functions? I do single maths so it’s not something I’ve done yet and I wanted to know if I should go do it on my own.

    1. Hi Temi,
      according to the PAT website that is not on the syllabus. I’m pretty sure questions have come up in the past that require knowledge of it. The syllabus has always been a bit loose in my experience so if you have time it can’t hurt to learn it – it’s in the year 13 syllabus anyhow so it won’t be too hard.

  33. Another question: are we still expected to know anything on magnetic fields?

    “Elementary electrostatic forces and magnetism; electromagnets, motors, generators”
    This part has been removed in the updated 2018 spec and there doesn’t seem to be any mention of magnetism except for transformers. Is it still worth knowing as there has been questions on it in previous years

    1. Hmmmm. It would appear that perhaps it is no longer required, perhaps the A-level spec has changed? However, questions that are technically off-spec have certainly appeared in the past; what is ‘obvious’ to a physics tutor might perhaps not be to a PAT student. It seems not impossible that things like F = ev x B = BIL could make an appearance, even if the specification says they ‘shouldn’t’.

  34. Basic knowledge of bodies in our Solar System, including planets, moons, comets and asteroids.

    What are we expected to know for this point? The planets is relatively obvious but I’m not sure what I need to know about the others

    1. So one thing that springs to mind is Kepler’s laws – there have been a couple of questions in the past that have involved knowledge of these. Other than that, looking at what has been asked in exams, I expect it means just knowing the planets and what order they’re in – basic general knowledge really.

  35. Hello M,
    Thak you for this beautiful site and also for replying every comments.Well can you please clarify me if I have to know the concepts of
    (1) Hyperbolic functions(like sinh,cosh etc) and their identities?Their derivative and integrals?
    (2) Fluid mechanics?
    (3) Semiconductors & electronics?

    1. You’re very welcome Sadman,
      Sorry for the delay in responding to you!
      Hyperbolic trig functions are currently not on the syllabus, and nor is any fluid mechanics or material on semiconductors. As ever, you can check the syllabus at https://www2.physics.ox.ac.uk/study-here/undergraduates/applications/physics-aptitude-test-pat/pat-syllabus . It’s not totally inconceivable that a question loosely around one of those concepts could arise, but all the relevant information would need to be given to you.

  36. hello,
    I thank you for such a beautiful initiative to provide very authentic solutions to the PAT questions. but I wanna know if you can suggest me any pdf books or any books on PAT-like problems or at least on any of the relevant topics like mechanics or graph etc etc.
    thank you very much.

    1. Hi Sadman!

      Thank you so much, that’s very kind. I’m afraid I actually don’t know any at an appropriate level. If I happen to come across something I’ll put a link up.

    2. I saw this on a youtube video on preparing for the PAT and the interview. The recommended book was Professor Povey’s Perplexing Problems. It has a bunch of physics problem that should increase your intuition and problem solving ability.

      1. Great recommendation! Prof. Povey is a great writer, those problems look appropriate. He’s an engineering professor, and the PAT is also used for engineering admissions so I trust him wholeheartedly to do that well.

  37. Hi, I was wondering if you’d be able to expand on what these points in the spec mean because I’m not sure what they are asking

    Response of a system to multiple forces (it seems quite vague, I don’t know what they mean)
    The meaning of the terms friction, air resistance and terminal velocity and how they can be calculated

    When I search for air resistance and terminal velocity formulae it comes up with some really long equations with lots of variables. Does that mean we need to memorise these?

    1. Hi Ryan!

      Honestly, I have no idea either, that is extremely vague. I think it means that you can do problems with force diagrams that aren’t just being pushed in one direction, but that’s kind of vague too.

      As for memorising complicated formulae, I should think not. The PAT tests how well you can think about problems using only A-level material, you shouldn’t have to do that. In principle, I think they could ask you to solve a differential equation for the velocity of a particle with a velocity dependent drag force? That is the sort of thing you’d cover in the first year mechanics course at uni though.

  38. Hey, are we supposed to know lorentz’s force and other electromagnetism topics such as impedances and biot savart law?

    1. I honestly have no idea.

      I just had a look at the syllabus on the website and it doesn’t mention this, but questions have definitely come up requiring you to know that the force on a charge in a B field is F=qvB. I’d guess this isn’t mentioned because it’s usually on the A-level syllabus? Additionally, I’m not sure exactly what you mean by Biot-Savart law – I think of the awkward vector calculus law on the wiki page, which is most definitely not on syllabus.

  39. I am a 13 student preparing to sit the PAT, and I have a question regarding the content on the test. The syllabus does not mention differentiation of trigonometric functions, yet on previous papers the topic appears- there was this question in 2017-

    Differentiate y = 2x cos x with respect to x

    I am unsure as to whether this content will be tested or whether this was a question testing content on an old specification. As I am studying A level maths I won’t study this topic by the 30th of October unless I do so independently. I would therefore be grateful if you could advise as to whether you think this type of content will be tested or not.

    Many thanks

    1. Hi Nial,
      I’d learn it just in case. From the point of view of the examiners, I’m sure they just thought ‘oh that’s trivial, they’ll know that’ without having thought about it. It’s really not too hard, so putting an hour or so into learning it could be worthwhile.

  40. 2018 PAT done. What do everyone think. Hard? Easy? I found the style of qs very different from the past few years.

    1. As someone who sat last year’s paper as well as this one, I would say the difficulty of the paper this year was less by comparison, however a lot of people said they were tight on time. I didn’t find that I was as tight on time this year as I was last year (but I’d mainly put that down to the fact that I had a calculator to perform mundane calculations, whereas in previous years you’d have to waste time doing that). All in all, I think the paper was fundamentally difficult (as you’d expect with an Oxford entrance exam), but there were no “standout” impossible questions like the last question in 2012 for example or like the binary star system question last year… what on Earth was that!!

  41. Hi, thank you so much for this website it has helped massively with my preparation. I was wondering what value of g should be used in this years PAT as we will be able to use calculators and since 2015 the value that should be used hasn’t been mentioned in the paper? If, like previous papers, the value of g that should be used hasn’t been explicitly mentioned – should I just use 9.81 as this is the most accurate value? Thank you 🙂

    1. Good question. I don’t actually know any better than you about what to do here. But if I were in your shoes I would: (1) read the front of the paper to check that they don’t explicitly say what to do; (2) if they don’t say what to do then use 9.81; (3) keep the answer symbolic (i.e. call it ‘g’) for as long as you can before substituting numbers in at the end, so that they can clearly see that you’ve got the working right and which value you have chosen to use.

      1. For the benefit of future readers: even on 2018 where a calculator is allowed, the paper says to use that g=10kgms^-2. Always make sure you read the information given to you, and if in doubt keep your answer in symbolic form for as long as possible.

  42. Do we need to know about magnetism and electromagnetism and the accompanying formulae, such as Faraday law and lenz law and such

    1. No I don’t think so. Have a look at the syllabus on the Oxford website. It talks about knowledge of transformers and how they work, including simple concepts like the relation of voltage to number of turns in the coil. But that’s the only thing I think you need to know about electromagnetism.

  43. Hi

    First off I would like to thank you for this invaluable resource you have made here; it has been so helpful when revising. This year’s exam is under a week away and whilst I feel well prepared I was wondering if you had any ideas as to what might come up?

    Best,
    Will

    1. No problem it’s a pleasure to help. I’m afraid I have no idea what will come up. Also it’s not really in our interests to speculate: if the guess is wrong we look silly; if the guess is right we look dodgy. Very best of luck with the PAT — if you feel good about your preparation then that’s half the battle.

  44. Hi there!

    Do you know whether compasses (the drawing tool) are allowed on the PAT?

    Thanks in advance for your response.

    1. Hi — sorry I honestly don’t know. Personally if I wanted to bring in compasses (and I guess a ruler) then I would just do it — it’s clearly not giving anyone an unfair advantage. But please don’t take that as implying any knowledge of or influence over the policy on my part — I really do know no more than you do.

  45. Hello,

    Thank you very much for this resource – it’s been absolutely invaluable and I really appreciate the time and effort that must have gone into producing something this comprehensive.

    I wondered whether you could advise me as to what I sort of revision activities you think would be most effective at this stage: I’ve been doing PAT preparation since the beginning of summer, and I’ve done all the past papers a few times (apart from the most recent one which I’m saving for a mock nearer the time), as well as most of the relevant Physics Olympiad questions and the problems on the Issac Physics website that’s mentioned on the Oxford Physics site. Are any other resources/preparation activities you would recommend, aside from going over the content again and revising the necessary formulae etc.?

    Thanks again.

    1. Hi Eleanor. Thanks for your kind words. It sounds like you are really well-prepared. So I would do two things: (1) go back through some of the really hard questions and make sure that you really grasp them (because the one thing you want to avoid in the exam is the sudden realisation that you don’t really understand some topic); (2) relax, rest and (although this may sound a bit odd) copy what athletes do by visualizing yourself getting the right answers to difficult questions as they come up — there is not much time in the exam and so it really helps to be in the zone during those two hours.

  46. Hello! Could you please tell me, taking into account the new syllabus, if I have to know these things:

    The equation of the i-v graph for a diode;
    Trigonometric substitution when solving integrals;
    Capacitor charging and discharging graphs and equations;
    Projectile Motion;
    Hard integrals;

    Thank you so very much for the work you put into this website and for the attention you give to us, students preparing for the PAT.

      1. Hi Tomas — sorry your previous comment came in a rush with a bunch of others and got overlooked.

        Before answering your questions, I think it’s important to bear two things in mind. First, remember the mark that will get you an interview is normally less than 60% so it’s not a disaster if you haven’t covered 100% of the syllabus because you can afford to drop a few marks if you are strong on the parts of the syllabus that you do know; second, I don’t really know the answers to any question of the form ‘is X on the syllabus’ unless X happens to be mentioned explicitly word-for-word in the syllabus, so take what’s written below with a pinch of salt.

        • The equation of the i-v graph for a diode;
        • Capacitor charging and discharging graphs and equations;

        I think ‘yes’ to both. (Covered by “Understanding circuit diagrams including batteries, wires, resistors, filament lamps, diodes, capacitors, light dependent resistors and thermistors.”).

        • Projectile motion

        ‘Yes’ definitely. (Covered in all the many mechanics points).

        • Trigonometric substitution when solving integrals;

        Probably ‘no’ I think. (Not explicitly mentioned anywhere).

        • Hard integrals;

        Depends on what you think a ‘hard integral’ is 🙂

        Finally, if you are short of time and you are in any doubt about what to cover, then focus on mechanics. This is the most likely class of problems to come up both in the tests and the interview, so it pays to focus on this.

        1. Hi!
          Thank you very much for the help. I have been doing past papers and BPhO questions and it seems to be going well.
          Thank you again for the amazing content on the webpage.

  47. Hi just wondering if the links to the different topics questions have been updated since 2015 as that’s the latest I’ve seen on most, don’t want to sound like im complaining I know it takes a lot of time to do this and it’s a very good resource

    1. Yes that’s a good point, I just never get round to it. I’m quite busy at the moment with work commitments but I will see if I can spend some time on it in the next week or two. Also if you, or anyone else, want to add some comments on the various sections then you can help make these pages better. If you think there are questions that should be added to a category than just paste the relevant links into a comment on that page, and then everyone can see them immediately, and I can pull them into the page text later.

  48. Hi, would you say it’s a little too late to prepare for the PAT thoroughly or is there still enough time to potentially get an offer? I still have A2 Maths & Physics content to learn so I haven’t actually started preparing for the PAT which seems a little daunting as it’s only 70 days away now… If I finished all the A2 Maths and Physics content by the end of September, would you say that leaves me with enough time for strict PAT practice or is a month still a little short?

    I’d also like to say, this seems like a very useful resource which is organised really well, so congratulations on making something which is so useful, lol.

    1. Hi — it is absolutely not too late to prepare for the PAT thoroughly. If you want to do well in the PAT then you should focus on that, rather than learning all the A2 Maths and Physics content. After all, your A2 exams won’t be starting until 7-8 months after the PAT is over. So your best bet is to learn the bits of A2 that are on the PAT syllabus and do lots of past paper questions on this material. The PAT syllabus changed a fair bit a few years ago, so when choosing PAT past paper questions, look here to check whether the subject matter of the question has not been dropped.

      1. Hi, I meant that I am still learning the A2 Maths and Physics content required for the PAT, not all of it. Sorry about the misunderstanding.

        Would you recommend I only focus on the A2 content and get it out of the way as soon as possible or do PAT prep alongside learning the A2 content?

        My plan is to do PAT and BPhO papers to prepare. Will this be sufficient or would you recommend I do other things on top of those 2?

        1. Ah gotcha — no I would recommend starting on past papers now and learning the extra stuff in parallel. The longer you spend actively doing the right kind of problems the better you tend to get at them. I doubt there’s much point doing anything more than PAT and BPhO problems.

  49. Hey, excellent solutions – helped me to mark internal PAT mock at school. Error on Q.6 though – C is the answer, but A cannot be the answer. Try graphing the equation. Limit to x= -3 from the left is negative yet graph is positive to the left of -3 asymptote.

    1. Also in Q.19, Cos.wt = Positive (Root 3)/2 only. NOT Negative (Root 3)/2.
      So two of the suggested solutions are not valid.

    2. Marco, you’re absolutely right — it’s really helpful to have errors like this pointed out, so thanks very much. Please post any other mistakes you find on the question where you find them. I copied your other spot to the question itself — thanks a lot for that too.

      1. No, thank you for setting this all up!

        Q.22 The radii of the circles are missing a half, due to dividing r^2 by the 4 that accompanies x2 and y2.
        36+16-43=9 so first radius is 3/2. Second radius, 5/2.

  50. I’m pleased to say that I received an offer for physics yesterday! Without this site it wouldn’t have been possible, so I would like to thank the admin(s) very much!

      1. Hi Sir,

        Due to your invaluable efforts in preparing me for the PAT and interview, I was offered a place for Engineering Science. Hence I would like to attempt the impossible, that is, thanking you enough!

    1. You shoud do yes, if you have covered C4 anyway. Im sure they’ll give you the formula in the interview if you forget though.

      Whens your interview? Whats it for?

      1. We haven’t covered C4. In fact just to do the PAT I had to self teach so much C3/C4 I basically don’t attend the C3 lessons at school now but I remember not learning integration by parts because it looked really chunky and it was the fun bit of C4 so I left it so there would be 1 thing (along with vectors) to look forward to.

        I am really worried now because it’s 100% true I was told we just started the chain rule in class so … I yeah … scared!

        Physics!

        1. I wouldn’t worry too much about minor details of the maths syllabus. Remember, it’s not an exam, it’s a face-to-face dialogue, so if you have never learnt some specific details then that will be obvious, and not too big a deal. The interviewers want people who will be able to do well on the course; this means they are are looking for general problem-solving skills and the ability to grasp unfamiliar things quickly. Given two candidates, one with great physics skills who has no clue about integration by parts, and one with weaker physics who knows integration by parts backwards, they will prefer the former every time.

  51. In your experience @OxfordPAT, does the interview tend to be harder than the average PAT exam or easier? Is it similar in difficulty but different in style? Are there any PAT questions you think are particularly like interview ones? Thank you.

    1. Hi Saiyed. The interviews are completely different to the exam in style, because interviews are all about interaction. In general, the more (sensible) questions you ask, and the more good points you make to the interviewers, the better. There will almost certainly be a whiteboard, or some sheets of paper, and even if you can’t immediately answer a question, the examiners will be interested in your ability to make progress towards the answer (e.g. drawing a diagram, writing down some key equations …). I haven’t had enough reports back to be able to predict the content of the questions, but they seem to be similar in difficulty. One useful exercise would be to review some of your answers to past papers and think about how you would explain the key steps verbally. Above all, in the interview just remember to relax and contribute as much as you can.

  52. Is it just me or was this years PAT more difficult than any other since 2006, the questions were from previously untested areas as well???

    1. well, you see i found multiple choice questions very easy, but last 4-5 questions IMPOSSIBLE to solve. in last 3 questions :
      paramatrics , sound wave and refractive index i couldnt wrote anything, srely loosing 27 marks. im expecting about 50+- 5 marks
      and also cut off mark to be super low

    1. Nope 😦 in the open day they said that you could be invited for interview as late as a week before when they are and if you don’t hear back by then the contact admissions

        1. The minimum notice is 1 week before the assigned interview dates for your course, which have been published on Oxford’s website, if you haven’t heard from them by then, you should email your college.

          1. Thank you! It says Physics Interviews are on the 11th, I’m surprised all the Physics interviews will be on that specific date. You’d think they’d spread them out.

            1. Is physics over 1 day or several days? Check the college website as well, they may have specified if you may be invited for an interview on a different day.

    1. Hi George. Have a look at http://www.ox.ac.uk/admissions/undergraduate/applying-to-oxford/interviews/sample-interview-questions. The Physics, Engineering, Materials and Maths questions are relevant. A lot of tutors (rightly in my view) think that proficiency in mechanics is a good indicator of potential to do well so you should make sure you are really comfortable with the mechanics syllabus (and don’t forget the simple machines, like levers, pulleys, capstans, etc.).

      1. hey sir,
        an energy problem related to catapult:
        A catapult as shown projects a particle on a smooth horizontal surface. a light elastic string of natural length 8a and modulus of elasticity 2mg is attached to two points Q,P at a distance 8a apart on a horizontal table. A stone of mass m is attached to the mid-point, at a point D, drawn back a distance 3a to a point C and then Released.
        what energy is stored in the elastic string when the mass is at point C?
        HERE is my answers: extension=10a-8a=2a E=1/2kx^2=0.5(2mg)(2a)=2mag the answer is 1/2mag which I am not agree with. what do you think?
        thanks

  53. Do you guys know how much your pat score matters if you do well in interview for the final decision?(marks and other school stuff being very good of course )

    1. Seriously why are you so worried? You just voted that you got the 70-80 bracket on the student room when you can clearly see that’s a very impressive score. The PAT is important, and if you have a good score then the interviews matter less (but they are probably the single most important aspect of the process).

      1. mainly because I don’t know how harsh they grade on small clumsy mistakes. I think I might have made some of those in questions i originally thought i got wright, and those add up…

  54. From what I remember, here is a list of multiple choice questions and longer questions from the test:

    differentiation
    sum of geometric series
    capacitor
    pulley
    graph of a function
    ball on the moon (it will go into orbit obviously lol)
    integrals
    momentum of an electron
    wavelenght of EM spectrum
    resistance
    logs

    binomial
    probability
    spring
    volume
    drag force
    planets
    sound waves
    parametric equations
    circle
    calculus
    refraction

    I am missing one multiple choice question; does anyone remember it?
    Also, let me know if these look right: https://imgur.com/a/mDrRY

    1. Hey do you know what the marks were for the probability, volumes, binomial expansion and springs were by any chance?

          1. Yeah, that’s what I meant. If the original equation was in the form 2x^2 + 2(a+b)x + 2ab, it could be factorised into 2(x+a)(x+b), and so the equation with equal roots was (x+a)(x+b).

            I actually couldn’t do the spring question. I tried linking energy stored to force on the mass, but eventually moved on.

            1. Cheers for the solutions. Are they for the ones you could do, what about the other questions? With the spring question, just resolve horizontally on mass m. If F(spring)=kx, where the x is the extension, then for the mass to move this force needs to be greater than the frictional force, so the first instant it moves is when Fr(mumg) =kx, then you rearrange to get x=fr/k. Thus the displacement of M from m is L + x=L+fr/k. Hope this helps.

                1. I wasn’t too sure, I think they would accept either. It’s just because they gave the length of the spring (L) so I thought you would have to include it in some way. As long as you indicated what kind of displacement it was, (I. E. from mass m or from its starting position) either with words or a diagram or evenvaguely made it evident, you would get the marks.

                  1. aah i though i messed up :), I did draw a forces diagram indicating direction but don’t know if that will suffice, I thought it was obvious that it will be displacement in order to expand the string

              1. In my working I drew a diagram and wrote mg(mu) = kx. I didn’t rearrange for x or state that this was the final answer (I thought the final answer would be in terms of M also). Do you think that this would be enough for at least 3 out of 5 marks?

                The solutions are for the questions I could do, or figured out after the exam. I don’t remember the specifics of other questions, and didn’t even attempt the final one.

                1. M doens’t have to be involved since it doen’t matter wether you are displacing a mass of just moving it with your hand, at least in terms of force needed to move m, work done on the other hand would involve M

      1. They may have been. I don’t remember the question exactly. I am certain that the top of each circle had a y co-ordinate of 3.5 and the tangent had length 8. And there are indeed supposed to be two additional diagonal tangents which I have forgotten about (does anyone know if they will also have a length of 8?)

        1. no they will have a other length but I only had enough time to do one tangens since i turned a 4 into a 2 somewhere in the process of deriving the equations for the circle so I had to do the entire question over again. I didn’t even have time to realize that root(2,25)=1,5. How many mark do you think you’d lose if your answer is a root that is equal to 8 but not neatly worked out?

          1. That’s suprisingly high, how did you find the test? I struggled, and couldn’t at all do the parametric equations question, or the sound wave one. The rest were okay, but I left a lot of the MCQs to the end and ended up having to guess at a few of those. Pretty sure I’ve got <30.

            1. Well I’m not quite sure, I’ll probably get less. I just tried to do all the activities even though I wasn’t sure about them, if they give me some points for trying even though I didn’t get the correct answer then I might reach up to a 50. If not, most probably I’ll get a 40.

  55. I think I speak for most people when I say we all struggled and were taken by surprise. But is this really such a bad thing? This particular paper that we just sat is representative of what the PAT should be about. It was about really getting to grips with an understanding of physics. The older papers, to some, started to become almost like an A-level exam, and became devalued – as seen by the very high “pass marks”. If indeed this is the new format for the PAT, then I think as the first year of those subjected to it, we must feel we that we were subjected not to something to our detriment, but instead about being oneself in the face of academic adversity – a challenge to you and me. This means it rewards insight far better than punishing deviation from a rote method. Those of you who could get an answer to anything, you should be proud!

    Of course our friend here Mr Oxford PAT will be here to guide the next batch of Physics wannabes, but I think if this is the course that the test takes, it is one we should be comfortable to acknowledge hereon-in; a difficult test for a difficult course.

    I hope everyone felt they performed their best and prepared as well as they could under the circumstances; we’ve done it now, and no matter how good or bad, you’re already miles ahead of your physics A-level colleagues. Well done everyone.

    1. I think this comment sums it up really. There were definitely hard questions in that paper, but the boundaries will be representative of this, so nothing different to usual years in terms of getting to interviews and offers.

      Now that everyone has sat it, can anybody list out all of the non-multiple choice questions? (Q13-23) I can remember 9/11.

      Thanks.

      1. Like you I can only remember 9 non-multiple choice questions, but hopefully the ones I can’t remember you can. In no particular order:
        – Two sounds waves of the same amplitude but different frequencies and wavelengths
        – Gold ring dropped in a tank of two transparent liquids with different refractive indices
        – Parachutist differential equation
        – Derivative with respect to t of an integral with respect to x
        – Cone and sphere of equal volume
        – Tangents to two circles
        – Comparing the speeds of stars in binary and triple star systems
        – Masses connected by a spring on a table
        – Parametric equations

        1. I cant remember but was the binary and triple stars problem considering the stars following the same circular path with radius r or was each star in a different path ?

          1. In each system of stars they all followed the same circular path of the same radius, and the radii of the two systems were the same; the only difference between the stars in each orbit was that they were evenly spaced around the orbit, so in the binary system they were always directly opposite each other, and in the triple star system they formed the vertices of an equilateral triangle.

  56. The PAT is done and dusted, and now all we have to do is wait for results. Thank you ever so much for this excellent learning resource and all the time and effort you put into it. If it wasn’t for this website I wouldn’t have even a remote chance of passing.

  57. What the hell was that? I had practised for months and I don’t even know if I scored a total mark of 30. Well, I guess my dream of getting into Oxford won’t become true.

  58. I’ve just un-Approved them and I’ll enable them again in 24 hours. Sorry for the delay — I just happened to see the comments in a rare idle moment at work …

  59. This was an insane paper. What the hell was that SHM question? And I had absolutely no idea how to do the parametric equations question. And who remembers the volume of a cone??

    I prepared a lot too.

    Any chance I had at Oxford is gone with that paper 😦

    1. lol i swear i had an awful struggle with volume of cone but i did it.
      i got 0/8 on parametrics
      i got 0/9 on sound waves
      i got 0/9 on gold ring experiment
      and maybe 4/9 on circles
      2/5 on spring masses
      but dont worry i think nearly all candidates will fail that 3-4 questions
      and i expect mean to be pretty low

      1. That’s pretty cool, I tried to derive it somehow but couldn’t!

        I did the same. I found the ring question okay though I think.

        Thanks for the reassurance! Mine is unsalvagable but I hope yours went well otherwise!

      1. Seriously. I did back to 2009 and it was absolutely nothing like any of them. 2013, 2014, 2015, 2016 were a piece of cake compared to this one, therewas only about 1 or 2 hard questions that I really struggled with, and about 2 or 3, 8 mark questions too. I think I counted 6 9 mark questions??

  60. this was the wierdest but not the hardest pat yet.
    parametrics, sound wave and water experiment were nerly impossible to solve.
    whats your thought guys?

    1. I thought it was the hardest pat till date, their was some difficult integral question as well. I also agree that those 3 were extremely difficult, and combined they were worth around 30 parks.

  61. Hey sir. I just want to say thank you for your great effort and all the amazing work you are doing now. No matter how the test goes in the end, I just want to say thank you!

    For the PAT people, good luck tomorrow and I hope you all do well!

    1. Just want to add to what you said George. Anonymous (I don’t know what else to call you!) you’ve been amazing at taking the time to do all this, and answering every single question posted by everyone. Thank you so much for doing it all, it’s a truly selfless thing to do. I really appreciate it, and it’s reassuring to know that people like you exist.

      Best of luck to you too George, and everyone else!

      Thanks again 🙂

      Nad

      1. Hi Nad. It is a real pleasure to be able to help, and I’m very glad it’s been useful. Thanks for your questions and comments: for every person who asks a question there are another twenty who were wondering the same thing but didn’t actually write it down, so every question is a contribution. Good luck for tomorrow.

  62. Hello! Thanks for all the help!

    do you have to know how to differentiate and integrate log? (logs in the form y= log10(x)

    1. The only result you get is whether or not you are invited for interview. In previous years (and I presume this year) interviews have been held in mid-December, after the end of term. So everyone is notified in late November / early December as to whether or not they will be interviewed.

  63. how long does exam session last?
    i recieved an e mail from the British Council that it will last 1 h and 20 min
    should i trust it?

    1. Good old British Council, always on the ball. I’m pretty sure it is 2 hours. If they try to stop you after 1 hour and 20 I would point to the big writing on the front of the paper.

  64. Hi,

    This website is amazing, thank you.

    I have one question though; on the front of the paper it mentions that we need to answer to 2 s.f. unless otherwise stated. However I often get an exact answer like e.g. in surd or in terms of pi. Does this mean I have to convert it to decimal? That would seem like an unnecessary waste of time…

    1. It’s a pleasure, glad it’s useful.

      You are absolutely right about answering to 2sf. In the old days it was only the physics section that said this, but now the sections are going to be mixed up I doubt they will be able to do that. So this year especially make sure you read the front of the paper and just do exactly what they say.

      I agree that taking, say, root 2 and writing it as 1.4 is a complete waste of time, and quite often I’ve broken that rule when writing down my answers. But if I were actually sitting the exam I would be inclined to follow it to the letter.

      1. Thanks for your reply; I just looked at the new specimen paper, and they now say to give your answer in simplest terms unless otherwise stated. That means no converting from root 2 to 1.4 hhahahaha. cheers

  65. can anyone please tell me what is format of exam like?
    will i have an extra paper to make calculations ? or will i have to write everything on the exam paper.

    1. I emailed them about this; you are advised to do everything in the paper because they can’t be responsible for lost sheets; if you do use an extra sheet ensure with your exam officer it’s securely attached and clear what it’s being used for (IE: write “x^2+2x+1=0 … continue on extra sheet”).

      1. Thanks for that. I also think that one of the things you should also focus on is making a neat and tidy answer — it is a big help to the marker if your answer is clearly laid out and not full of crossings-out. If you do a tidy answer there is (almost) always enough room.

        This sometimes means having a bit of rough paper to do calculations and sketches on. In my experience rough paper is provided by the local site that is hosting the exam, and it should always be available. The fact that the examiners are talking about what you should do with extra sheets that you want to hand in implies that they expect you to be able to get hold of extra sheets if you need them. If you’re in any doubt, check that your school/college will be providing them.

    1. Just integer powers, although it may be useful to know the formuls for fractional powers as it can be easier than the binomical expansion.

  66. Thanks for all the solutions, they have really helped.

    I was wondering whether they could ask us to use archimedes principle and elements of fluid dynamics in questions. I know in the past several multiple choice questions have required you to apply it , however could, for example, a non-multiple choice 4 marker come up? I think some of the problem solving questions have required you to understand the relationship between how much objects float and thus how their densities are related.

    I am a little confused by the fact that the formula for the refractive index of a medium is v/c, where v is the speed of light through that medium and c is the speed of light through a vacuum, as the refractive index of a material changes as per the wavelength of light that travels through the medium, so how can the formula simply say ‘light’, does it mean of ‘the specific wavelength’.

    I was also wondering whether you would be able to post the solutions to the 2001 and 2002 ‘PAT’ papers (they only had maths back then), I would really appreciate it. The link can be found below.

    https://www.whatdotheyknow.com/request/request_for_physics_aptitude_tes

    Apologies for the lengthy comment.

    Thanks.

    1. I’m afraid I just don’t know the answer to the first question — but I guess it is possible that you could get a question about Archimedes’ principle.

      To answer your second question, yes, the refractive index of a material can change (a little bit) depending on the wavelength of the light, and the speed of the light through the material changes accordingly (because c doesn’t change).

      I’m really sorry but I won’t get time to do the answers for the 2001 and 2002 papers for several weeks because I’m quite busy at the moment, so any answers I do will come out some time after this year’s PAT.

    1. I think you really only need to know about SHM. But it certainly helps to know the differential equation of SHM, the force/distance graph that corresponds to SHM (see e.g. Q22, 2012), and the how the differential equation is solved and the frequency calculated.

        1. capacitance= Permittivity of dielectric x Area of overlap / distance between the plates
          i would recommend you look at the old (1995) A level books, everything in general is discussed best in there.
          + do not forget the charge and discharge graphs of capacitors

    1. Good question — I’m not sure. Have a look at previous questions — as far as I can remember nothing has come up which doesn’t just count as ‘general knowledge’ (e.g. inner planets, asteroids, outer planets, er, that’s about it).

    2. how does the number of moons change.
      how does the length of the day changes.
      how does the length of the year change.

      u dont have to know exact numbers but need to have a basic idea

  67. hello, will it be helpful to study m2 and m3 books? i mean those topics which are written without integrals in physics like statics of a rogod bodies

        1. I wouldn’t worry too much about this. You are in the same boat as everybody else; the only thing anybody knows is what is written in the syllabus and the questions that have been asked previously.

  68. Hi, For physics would there be any question from the topic of properties of matter, questions such as the speed of the particle 3/2KT or questions which releate to pressure, volume, temperature equations

  69. When finding the area between function 1 and 2, can you get take the the upper function from the lower function, without worrying about some of the area being in the negative y part of the graph.

    1. You mean do you need to know something about summing series? Yes: there have been various questions that are covered in the syllabus under the general banner of Arithmetic and geometric progressions.

  70. Dear Anon

    Something I find really odd is thst in the syllabus they removed all references to radioactive decay including half life but yet in the 2017 sample paper, there is a question on half life in the multiple choice section! Can you possibly shed any light?

    Thanks

    Nad

    1. No I can’t — but I guess the example paper goes through less of a review process than the actual exam, so this could have squeaked through. I would still advise that you only revise what is in the syllabus.

  71. Hi, How much trig do we need to know? Do we need to know about cot, cosec, sec and their derives and things? And solving equation using them

    1. Gravitational force is actually proportional to the mass involved, relative to the other; in other words, it’s mass multiplied, not added. That’s why if you have an object twice as “heavy”, it’ll experience twice the force.

    2. Thank you for that Jess. And it’s great to see a PAT student from last year starting off at Oxford. I hope the course goes well and you have a fantastic time.

      1. It was a pleasure, thank you for everything you’ve done on this site! It was really helpful for me to prepare for the PAT. The course is going great so far, thanks for all your help!

          1. No problem, I always got stuck on the gravity questions too!

            The first equation links Force F with the universal gravitational constant G, the mass of one orbiting object M, the mass of the other orbiting object m (it doesn’t matter who is orbiting who because defining that depends on your reference point, and this equation is universal so doesn’t care about reference points), and the radius r. As Seb Wilkes said above, gravitational force is proportional to the mass involved and inversely proportional to the radius squared.

            The second equation comes about when you assume that the orbit is circular so that you can equate the first equation for force with the equation for force in circular motion, F=mv^2/r – you should get GMm/r^2=mv^2/r. A bit of rearranging gives you the second equation, which links velocity at a point in the orbit, v, with the mass of the object at the centre of the orbit, M, and radius, r. This equation is interesting because it proves that the velocity at a point in the orbit does not depend on the mass of the orbiting object!

            The third equation is derived here: <a href="https://oxfordpat.wordpress.com/circular-orbits-under-gravity/"https://oxfordpat.wordpress.com/circular-orbits-under-gravity/

            The last equation uses the definition v=ωr and the third equation.

            I hope this was helpful! Please do comment on my blog if you have any more questions so that the explanations can be found with the equations.

            1. I still can’t help but feel a bit confused though because in class, in our end-of-topic tests (and from the teacher for that matter) I’ve never once seen this version of the equation. I don’t dispute it, obviously, but might it be a tad overkill for the PAT?

              Just to clarify then, is it the reason you add up the two accelerations because it’s like “relative” acceleration (where it just happens to move towards each other here, right?). If that’s true, I guess that means when I’m falling after a jump, the “actual” acceleration is not just “g” but also g+(Force experienced/Mass of Earth)?

              1. Basically yes. If one of the masses is much bigger than the other then the answer is almost identical to the one you were taught. But it’s always worth being exact in your derivations because that way you avoid the nightmare scenario of suddenly asking yourself a deep and subtle question in the middle of the exam.

  72. Hey sir. Do you have any advice about final phase of preparation of the PAT? I have done most of my PAT past papers ( left one for the final mock) and AS challenge papers and are currently unsure what to do next.

    Thanks,
    George

    1. Blimey, you’ve done a lot more than most people I’m guessing. So I would look over the questions that you’ve already done, and just check that you are really confident with the material that they cover. After that I would take it easy 🙂

      1. Dear Anonymous

        In response to your comment, won’t most people have done exactly that and prepared for it as such? I’m assuming that anyone who applies to sit it will be relatively serious about doing so, and thus have prepared. Just curious!

        Nad

        1. Hi Nad. Well, judging by the traffic patterns on this site, quite a few people started last week.

          [To anyone: If you are reading this comment and you’re one of those people, don’t worry, there’s still plenty of time if you work reasonably hard over half term.]

  73. Hi, do you have any good sources to learn more about pulleys, such as the ones that come up in q15 of PAT 2014?
    Thanks

    1. I don’t really. But, pulleys are really easy because you just need to remember one thing: in a frictionless pulley, the tension in the string is the same on either side of the pulley. So, in a complex system of pulleys, the tension in the string is the same everywhere. If you look at some questions with this in mind, this should help.

      (BTW I’m pretty sure you will only get asked about frictionless pulleys, but when you’ve mastered them you should be able to work out what friction will do).

  74. Hey sir just a random question about statis: a student stands in the middle of a balanced plank which sits on rollers on top of a column.no friction between the plank and the top of the column. If the student walks to the right, smoothly , what happened to the plank and the balancing of the system on top of the column.

    The answer is that the plank will remain balanced and the central of mass remains in the same place. I agree with the first part of the answer. However, i thought the plank will move to the left to balance the weight of the student. I don’t understand why the centre of mass stay the same.

    Thanks

    1. Hi George, thanks for your question. The answer is right, but it’s not stated clearly enough, and I think it’s just the lack of clarity that you are finding confusing.

      When there are no external forces acting on a system its centre of mass won’t move. But the ‘system’ here is the plank plus the student. Suppose the student has mass M and the plank has mass m: if the student’s position is moved a distance x to the right, in order for the centre of mass of the system to remain stationary, the plank must move Mx/m to the left (so in fact the student will have travelled x(1+M/m) with respect to the plank but since the plank has travelled -Mx/m with respect to the column, the student only travels x with respect to the column).

      So yes, ‘the plank will remain balanced’ because the centre of mass of the plank and student combined will not move.

      If you want things to be stated absolutely formally, a more formal way of stating the fact that “When there are no external forces acting on a system its centre of mass won’t move” is to consider momentum. Force is the rate of change of momentum with respect to time. In this case the force is zero so the momentum must be constant, and since the plank+student were stationary at the start, their combined momentum must always be zero. So at time t, when the student is moving at velocity v(t) to the right, the plank has to be moving with velocity u(t), where u(t) = -Mv(t)/m. Distance is the integral of velocity with respect to time. Since -M/m is a constant, you can see that the distance travelled by the plank is -M/m times the distance travelled by the student.

  75. Hi, just wondering if density and force due to density in water and all those things are needed in the new syllabus. Thanks 🙂

    1. Yes I think you do. These exams become much easier if you know that distance is the integral of velocity with respect to time, work done is the integral of force with respect to distance, force is the differential of momentum with respect to time, and so on.

      1. thx for reply.
        Btw u have any sources or any books with i can order/ look up, in order to learn inequalities and curve sketching in a bit more detail?

    1. No I don’t think so. To do well on mechanics in the physics section it helps to understand the differential equations for exponential decay and harmonic motion, and to understand how to add and subtract vectors. But there’s no need to understand anything more about differential equations, or to know anything about dot or cross products, matrices, or anything else that might be regarded as doing maths with vectors.

        1. Ney mate.
          That was a simple chain rule from last chapter of C3 and 4th chapter of C4.
          I recommend u working on that, because, as it seems, not many have completed it successfully, so i expect to see the same question this year. and btw don’t forget about circle and differentiation of circle equation.

          1. Yes that’s right. I like this question because you only need to know one fact, that du/dx = du/dv dv/dx, but you also have to be comfortable using that fact and following it through to its logical conclusion.

  76. Hi, Just wondering if there is anything that is not in the A and As level (AQA) maths syllabus that will be in the exam.

    And thanks for all the help on the website, you are legend!

  77. Hey,
    In your opinion, if I am pursuing an engineering degree and plan to go to graduate school, is it better to go to another undergraduate school and then apply for an oxford grad school, or does going to oxford undergrad give me a bigger advantage at getting accepted for oxford grad school? Money is also an issue that I have to consider so I was wondering if it is worth going to a cheaper undergrad with less debt or is the oxford undergrad degree worth it enough so that I should take a job for the next couple of years to pay of its debt and then try to get into oxford grad? This is all hypothetical of course but I honestly just need some deeper understanding of what I should do

    1. I’m afraid I really have no idea. Ultimately in your engineering career it doesn’t matter that much where you studied, but this site is just for people who have decided to do the PAT and want to do well — I’m not an expert in different university options.

  78. this is probably the wrong place to post such a thing, but I just want to thank you for taking personal time to make this great and free website. You tirelessly answer questions, they are also always presented logically and clearly! for whatever reason your doing this, i just want to say thanks and keep it up!

    1. Thanks for the kind words Tony. I was lucky enough to have a top-class education for which I paid nothing and this is an opportunity for me to put a little bit back. I know how much effort and commitment the candidates put into the PAT and it’s just a pleasure to be able to help them out.

  79. Hi

    I just have a general question about what exactly “engineering science” is..

    So let’s say I am blessed with the extraordinary opportunity to study engineering science at oxford, if i come to the US do I need additional qualifications if I want to get a job for engineering? Like as in do I need to re study some of the material? I feel like the UK definition of what engineering science is and what the US definition is is different but honestly, I have no idea. I’m just a bit perplexed..

    1. I’m not an expert on the details of the qualifications, but I’ve worked with UK and US engineers and the basic content of engineering courses doesn’t vary that much; I think that a good degree in engineering science from Oxford would be highly valued in the US. Of course in most engineering disciplines you will do some more specialized training after your degree anyway, whether it is via academic or industrial courses, so all hiring managers are looking for in your bachelor’s degree is (most important) that you did well and (second most) that the university has a good reputation.

      1. One more thing–do you know if oxford super scores SAT because in the international qualification section of the cite it doesn’t specify. Would you happen to know if the 1400 on Math and Reading + 700 on Writing is for a single exam or is it a super score criteria? Thank You so much

  80. This website is a Godsend haha 🙂

    Just a specific question–is knowledge about thin films recommended for the PAT? I understand air wedges and single/double slit interference but just have a hard time for some weird reason getting my head around thin films and bubbles!! xd

    Thank you

    1. I don’t have any special or official information. But there has never been a question on thin films, and the topic is not mentioned on the syllabus, so I think you would be best off ignoring it and concentrating your efforts elsewhere.

      Of course, thin film effects do just result from reflection, diffraction and interference, so it’s not impossible for a thin films question to come up, but if it did I would expect that it would be posed in a way that enabled people with no specific knowledge to work it out.

  81. Hi there! Great resource but you already knew that 😉

    I’m in the early days of revision. Looks like simple harmonic motion is one of the past-paper questions but that’s not on the spec? I checked the changes to the spec but couldn’t find any mention.

    If I have any more questions can I keep asking? Thanks anyway 🙂

    1. Hi, glad you think it’s useful. Ask as many questions as you like.

      Simple harmonic motion has come up quite often over the years. I guess the examiners would consider that a full understanding of “distance, velocity, speed, acceleration, and the relationships between them. … Response to forces; Newton’s laws of motion; … Springs and Hooke’s law” would cover cases like SHM even though it is not explicitly mentioned. For example, I think the phrase above covers knowing that velocity = d(distance)/dt, acceleration = d²(distance)/dt², and force = mass x acceleration; given this knowledge, plus Hooke’s law, you can derive the differential equation md²x/dt² = -kx for a mass-spring system. It’s not that hard to solve this equation and “knowledge of elementary mathematics will be assumed”. So while there is no mention of the phrase, I think questions which touch on SHM are consistent with the syllabus.

      In general, if a topic has come up before and it is consistent with the new syllabus, then it’s worth knowing about it. It’s certainly true that some topics have explicitly been dropped from the syllabus since 2014 (e.g. nuclear physics) so there’s no point revising them, but, as you say above, SHM isn’t one of them.

  82. Hey

    How in depth do we need to know in analyzing circuits? I was briefly exploring the topic and found a a wide variety of topics that I wasn’t aware of, such as Wye Delta Transformations, rotational symmetry and node and mesh methods for simplifying circuits. Would it be safe to say that these in depth aren’t required in the pat?

    1. Yes it would be safe to assume that you don’t need sophisticated circuit analysis techniques. Certainly you won’t need to know about Y-Δ transformations or node/mesh methods (the former they would have mentioned in the syllabus, the latter are more suited to writing a program to solve the circuit). Rotational symmetry you won’t be allowed to know about, but it is useful to understand how you can use symmetry to find places where the current in a link is zero. See https://oxfordpat.wordpress.com/oxford-pat-2016-question-20/ for a good example.

    1. Hi Raafiul, thanks for your question.

      No, you don’t need to know anything about thermodynamics or fluid mechanics for the test. It’s important to look at the syllabus and make sure that you cover it (and don’t spend too much time covering things not on the syllabus). Also, be aware that the syllabus has changed so some past papers questions will not come up again. Have a look at this page https://oxfordpat.wordpress.com/changes-to-the-syllabus/, which has the syllabus and describes what has changed, and what you therefore don’t need to worry about.

  83. Hey ! I would like to say thank you! You have played a huge part in getting me an offer for engineering at Oxford ! Will definitely recommend this to aspiring applicants. Thank you again. May God bless you.

    1. I’d also like to give my thanks – your website was extremely helpful for me and I found out on Wednesday that I’ve been offered a place to read Physics! I can’t thank you enough for your help, it really has changed my life for the better!

      1. Thank you very much! It will go up in a couple of months, after the whole admissions process is over, so that I can get all my pieces together. The test went well enough thanks to your helpful answers!

    1. Honestly I thought it was alright compared to some of the other papers? There weren’t any hard graph sketching questions (although I did miss some questions because I didn’t study those parts, haha)

      1. Trust me, you wish that there were more graph sketching questions. The way that graph sketching is taught in school is not great, it just gets you to mechanically learn them instead of figuring them out.

        If you can factorise and differentiate, then you can graph almost anything with the right method! If you are interested, I can send you a pack that teaches you how to use a sign table. They’re quite simple and you might end up using them in your a level maths exams.

        You should learn how to sketch graphs for an eventual interview anyway as I’ve been told it’s a ‘go-to’ maths question. And if you can sketch confidently it kind of makes you look like a badass 🙂

              1. This is material that my teacher gave me so I can’t source its origin. Though, I think it is covered by fair dealing within fair use. (Criteria for fair dealing: The copy is made for the purposes of research or private study. The copy is made for non-commercial purposes. The source of the material is acknowledged.)

                If you feel this is unsatisfactory, by all means, do remove it. I’m just trying to help.

                1. I know you’re just trying to help and I don’t want to remove it. I just want to make sure that if somebody objects (e.g. the copyright owner) they have a mechanism for contacting me so it can be taken down. This ensures that they can’t come after me…

            1. Hey Emanuel thanks for the link! I was wondering if you knew the textbook that came from because I’m really interested (applying for PAT next year). Thanks in advance

    2. THIS YEAR WAS SO DIFFICULT hahahaha, the Math was actually a bit tricky and what was the last question in Physics😂😂😂, the last question for physics and the Octagon question for math went over my head😂😂

  84. Hi! I was wondering if it’d be possible for us to download the whole solution sets to papers all at once? I think that’d be very helpful! I’ve collated the ones I’ve downloaded (2010, 2014, 2015, 2015 sample), if you’re agreeable I could post them here/invite you to the google drive so that you could upload as well? (And maybe include a link to it in the front page)

    1. Hi Beth — thanks for your suggestion and your offer to share what you’ve done.

      I’m not convinced that it’s a good idea to have collated answers in files, because:
      (1) files get copied — currently if I need to change something (e.g. a mistake) I know that there won’t be incorrect copies circulating, because everyone gets the answers from a single server;
      (2) I want to encourage people to comment and ask questions on the individual answers and this will be less likely if they are available in one big file (for every person who asks a question I guess there are 50 who had the same doubt but just didn’t ask, so the answers can be disproportionately useful; and the easier it is to make a comment, the more likely somebody is to correct any error I might have made);
      (3) it’s useful to me to get statistics on which answers and other pages people actually look at because it is closely correlated with what people find difficult (e.g. in 2016 the second most popular question viewed was #22, 2012, which is no surprise because it’s very hard, but the most popular was #21, 2015, which I thought was really easy but actually has caused many people problems);
      (4) the first thing many people do when they get hold of a file is print it out. At a rough estimate, given the number of visitors to the site each year, this could result in about 300,000 sheets of paper being printed every year, which would be about 30 – 40 trees a year.

      Having said that, I’m always interested in other points of view, and I’m happy to be convinced or to find alternative solutions if you explain in more detail the value of having collated solutions.

      1. (1) I’d agree that it’d be easier with only one main copy. Although, I was planning on handing over access to the file server to you after I’ve uploaded what I have, so that you’d be able to update the files.

        (2) I meant the downloadable version as a supplement to the website. For example, if there was something I wasn’t sure about, I’d come back here to check the comments.

        (3) True, this can’t be replicated!

        (4) I haven’t considered this. I rarely print out my files; downloading is more to avoid the hassle of going online again to look for it. In this case, it was for last minute studying right before the PAT on the train where there was no internet. But I suppose more people would want to print it out.

        Honestly the main boon would be convenience. I’m probably not the only one who has done this and it’d be a big time saver for us users. But now I’m also worried about the trees, haha. Perhaps it won’t be such a good idea after all.

        Thanks for the detailed reply, and also for uploading the answers in the first place! They really have been very helpful.

        1. You make a good point about being able to study where there is no internet, so I’ve made a local copy of the site that it’s possible to download to your machine and use offline. It’s in this (116MB) zip file. I’d be really grateful if you’d give it a try and let me know if that would work for your use cases: if so I can just occasionally update the copy.

          Incidentally if you find yourself having to work without network access a lot, then you should get hold of wget ((this is the windows version, but it’s available on everything). It’s a tool for downloading entire sites — I used it to make the folder on the google drive. Takes a little bit of learning (e.g. see https://www.gnu.org/software/wget/manual/html_node/Advanced-Usage.html) but very useful …

          1. Oh wow, I never knew such software existed! Will be handy the next time I need to study without internet. Thanks for the link!

            I just downloaded the pack and checked it out, yes this works very well.
            Some minor things:
            1. The order for some of the questions is a bit scrambled in the oxfordpat folder (e.g. 2009; and for that year the folder for question 10 actually also contains 11 and 12, despite the name)
            2. When looking at the files in “oxfordpat.files” themselves they are sorted by date of upload (probably a wordpress thing) instead of by year of paper, but just searching for the paper I want and then sorting by name fixes it.

            Again, these are really minor things, so it might not be worth the time to fix them especially if you’d have to refix them each time you upload. This is just a heads up for anyone who wants to use the offline version! (Perhaps the link to the folder could be put into the main page so that people wouldn’t need to look through a comment thread?)

            Overall, it’s exactly what I would have wanted, thank you so much!

    1. Nah, I think this was really one of the hardest ones. So don’t give up too quickly… 😀 I hope to have 50 at least 😀

  85. Thank you for all your hard work and time that you put into these solutions. Hopefully tomorrow goes well for all of us

  86. Thank you so much for this invaluable resource – It has been vital to my preparation for the Pat so thank you!

  87. Thank you so much for the help and support that this website has to offer, honestly motivated me a lot and has provided an excellent resource to prepare myself the best I can during a short period of time! Honestly really appreciate it! And good luck to everyone tomorrow!

    1. I’m really pleased to hear it. Confidence especially is really important in this test. A very few of the questions are really hard, but quite a few are easy. The more confident you are the better you get at spotting the easy ones. Good luck!

  88. Whether tomorrow goes well or not, I’m glad I’ve learned so much new physics from all these solutions. Wish our Advanced Highers would cover all the pulleys and mechanics stuff! Cheers for the help.

  89. You are making me emotional! 🙂
    Thank you for all the help. I do have one question – how do you find the time to do this? I imagine you have a degree/doctoral thesis/job/family also. It is absolutely amazing that somebody has gone out of their way to do so much for others.

    Do you have a PayPal? I think I owe you a beer.

  90. Hi I would just like to say thank you very much for your tremendous help and efforts!!
    I was wondering if you had any advice for final preparation?

  91. Have questions regarding MCdeltaT formula been removed from the syllabus?
    For example the 20 marks question in 2013?

  92. Hi! Please could you take a look at question 14 from the 2006 paper? I have managed to work out the velocity (I think it’s correct) which is sqrt(2PT/M). It’s now asking about acceleration and I used V = u +at and substituted into the kinetic energy equation, where U = 0. My answer turns out to be sqrt(2P/MT), but looking at various solutions, the correct answer should be sqrt(P/2MT). Thanks! I really appreciate the work you’ve put into this site, I would not have been able to do many of the questions without it!

    1. You are absolutely right that the velocity is √(2Pt/M). But the equation v = u + at only works for constant acceleration, so you can’t use it here.

      Instead you need to consider the more general definition relating acceleration a with velocity v, which is that a = dv/dt.

      If you differentiate √t you get (1/2√t), therefore if you differentiate √(2Pt/M), you get √(P/2Mt)

      In the next part of the question, working out the distance travelled (call it s), you need to use the definition v = ds/dt, therefore s = ∫v dt.

  93. Hi I was wondering if it’s okay to leave our answers in terms of fractions/ or surds. Or do we need decimal values?

    1. The front of the paper states “Answers in Part A should be given exactly unless indicated otherwise. Numeric answers in Part B should be calculated to 2 significant figures. Use g= 10ms-2”

      Therefore, if it is maths and it’s a surd, it’s a good idea to keep it exact. If, for example, it is asking for the displacement as part of a suvat equation (unless it states otherwise) 2 sig figs will do.

      I know, it is quite annoying to get to the end of a meaty algebraic question to then be bothered by a long division.

      1. Exactly right. Note that sometimes I haven’t bothered to calculate the numeric answer in answers to Physics questions, and strictly you should do this, so that e.g. 2pi becomes 2*3.14.. = 6.28.. = 6.3 to 2sf. On the minus side, this looks a bit stupid and takes some time; on the plus side, it’s only 2sf.

  94. Hello, am i meant to study angular momentum it’s not in my a2 syllabus neither in the syllabus

    Do you have any advice for the last couple of days and what we should be doing?

    Whatever i score i am very grateful to you for this site, you are a blessing to those of us who could not get good help

    1. I am not an expert- I’m taking it this week too, actually!- but in the last 4 months during which I’ve gone through every paper and had a look at the syllabus I have not seen angular momentum at all.

      1. Yes I think you’re right. I wouldn’t worry too much about angular momentum. I think probably the only thing that might come up related to angular momentum is a variation on the classic ‘ice skater’ theme: when ice skaters spin with arms and legs stretched out they rotate at a certain angular velocity and when they pull their arms and legs in tightly they spin much faster. This is because their angular momentum must be conserved.

        The equivalent in the sort of systems that might be covered in the PAT is a single point mass spinning round on the end of a string. If the string is shortened then the mass will rotate faster. This is an example of conservation of angular momentum but you can explain the behaviour by just using your existing knowledge of conservation of momentum. When a mass is spinning on the end of a string of length r with angular velocity ω, it moves with speed ωr perpendicular to the string. Imagine reducing the length of the string somehow while still spinning the mass; no force is acting parallel to the velocity of the mass, so the momentum of the mass can’t change, and so its speed has to stay the same. This means that ω has to increase when r decreases.

        I think that’s it as far as angular momentum is concerned. I’m pretty sure you won’t need to know anything else.

  95. Hi,
    I really am happy I found this website, it is super helpful! I just had a quick question in terms of a topic that might come up. In the new specimen paper I’ve seen an example answer for a physics question using the application of how time period of an oscillating mass on a spring isn’t affected by a different value of surface gravity. This comes up in Simple Harmonic Motion in my A2 textbook, so I was wondering if we’d have to be well documented on the rest of Simple Harmonic Motion as a topic for the PAT? I’m not sure if I’ve missed seeing it in the spec! Thanks in advance!

  96. Hi do you think we need to know about moment of inertia and angular momentum? I think it once came up in a 2006 paper. Thanks!

    1. I think you should know that angular momentum is conserved, but I don’t think you will need to know much about moments of inertia for the PAT. If there is any reference to moment of inertia I think it will probably be explained.

  97. Hi!

    Could you tell me whether we need to know about Mechanical Advantage? I’ve noticed the PAT likes to use really complicated pulley systems, and I’m still struggling to understand the question from the 2014 paper on motors and Pulleys. To what extent do we need to know about pulleys?
    Thanks!

    1. Also, do you have any specific tips on what to do once you have 2 separate inequalities, and they must satisfy something like -2<f(x)<5
      so that it works for both?

      1. If you are solving two (or more) inequalities then for each inequality you will get a range of values of x that satisfies the inequality. If x satisfies multiple inequalities it must satisfy the conjunction of all of the individual inequalities; each inequality defines a single range, so to find the values of x that satisfy the conjunction of the inequalities you need to find the intersection of all the ranges.

  98. Hello, I’ve covered most of the syllabus except a few things under optics especially on elementary properties of lenses and prisms. What ‘elementary’ properties should I cover on? There was a question regarding Snell’s law in 2014, is that required?

    I’m more comfortable with reflection of plane mirrors but I might have missed a few key details, do you have any tips/resources leading to any possible reading on this?

    1. Good question — I think you probably should know about how real and virtual images are formed by lenses, and also Snell’s law (this came up and I don’t think any changes to the syllabus have removed it).

      As far as plane mirrors are concerned, it’s a shame that there has hardly ever been a PAT question on this subject so there isn’t much to revise. I suggest that you look at this page (http://www.physicsclassroom.com/class/refln/Lesson-2/Other-Multiple-Mirror-Systems) and make sure you can derive the relevant answers and understand the technique.

  99. Hi,
    Awesome website, specifically what calculus techniques are needed? The syllabus says integration of polynomials but I have already seen that further differentiation and integration methods have been used? Which ones are needed, product, quotient, chain? Will I need integration by substitution etc?

  100. Hey I know this is a stupid question but do you have any predictions for the PAT this year?
    Also Where would you reccomend people to focus on and how can I get extra maths practice I started soem STEP papers but they are significantly harder then the pat I believe

    Same question for how to study for physics ( I struggle a lot here, I think they reallt overwrite some of the questions and I lose track, quite funny actually

    Thank you for the site!

    1. Predictions for the PAT this year? Well we can all see into the future a little way, some further than others. But I’m afraid my foresight doesn’t even come close to covering the decisions made by people who set exams. My one prediction is that the questions will definitely only be about topics covered by the current syllabus, so make sure that you look at the ‘Changes to the syllabus’ link above to ensure that when you go through old papers you are not revising for questions that no longer have a chance of being asked.

      On the maths questions, I would cover all the basic topics above (Geometry, Algebra, Calculus, Logarithms, Series, Graphs, Probability) and make sure that you have really understood the answers to the past questions for each one. If you have a gap in your knowledge, use a search engine or a textbook, or ask a question on this site and I can point you to some resource.

      On the physics questions, you are right: many of the questions are quite wordy and describe some complex situations. I think this is on purpose: one of the key skills for a physicist or engineer is to be able to cut through a vague mess of words, spot the real physical situation being described, and produce a simple summary by drawing a diagram or writing down some equations. So if you are having trouble reading the questions, practice this process: try to create a diagram or write down the relevant equations, and then check back against the text of the question to see whether your simplification has actually captured what the examiner was saying. If you do this enough, you will get much better at it.

      1. That is some solid advice! Thank you I will do what you’ve said
        What did you mean by questions that can’t be asked which ones do you think won’t be asked? I’m sorry if I’m being a little dense again

        May I ask what your field of study is and where? Your site and solutions are great very nice and detailed and easy to understand so thank you

        1. Here is a good example of a question that could not be asked in this year’s PAT: https://oxfordpat.wordpress.com/oxford-pat-2011-question-24/. It is all about alpha, beta and gamma radiation, which has been taken off the syllabus. So make sure you look at the othr changes to the syllabus and ensure that you are studying things that are still covered.

          I’m glad you like the site. I actually studied Engineering at Cambridge, quite a while ago. In those days Cambridge had an entrance exam similar to the PAT but with three papers. I was very well-coached by some excellent teachers, and was lucky enough to get straight As, and an entrance scholarship. A few years ago I realised that not everyone is lucky enough to get the level of coaching I got, and that’s why I set up this site.

  101. Hi, thank you for your help, I found it really helpful for the maths section and now I’m doing the physics one and I have some questions about the program I should cover for the test. In the official syllabus it is said that is required a knowledge about “Elementary circuits including batteries, wires, resistors, filament lamps, diodes, capacitors, light dependent resistors and thermistors” but since I’m studying all this stuff by myself I would like to know which aspect I should see about diodes, light dependent resistors and thermistors. Do I have to know everything about them or it is enough a superficial understanding?

    1. That’s a good question. I think that for LDRs and thermistors, you should just know their basic characteristics (resistance dependent on incident light and temperature respectively) and have a look at a response curve for a typical example of each (i.e. how the resistance varies with the input light intensity or temperature). Diodes are much more important components and the examiners, being practical people, will therefore care more about them and set more complex questions on them, so for diodes it is worth having a more detailed grasp of what they do. A good example of a (pretty hard and subtle) question about diodes is https://oxfordpat.wordpress.com/oxford-pat-2014-question-16/.

      For the other components that the syllabus mentions, you should know Ohm’s law and how to use it, understand how to make calculations where batteries have internal resistance (treat as a battery with a resistor in series), and you should know that Q=VC, and I=dQ/dt, and be confident with the differential equations for capacitor charging and discharging that result from those two equations. If you know all that, then I think you will know everything you need for the PAT exam.

      Notice that you don’t need to know anything about inductors for the PAT, but if you want to also learn about inductors and LC circuits you will be at an advantage because this will help your understanding in general (more differential equations, the parallel between mass/spring/drag and inductor/capacitor/resistor, resonance, and lots of other stuff that makes the PAT look easy).

      1. Thank you, I hope I’ll manage to do it all by myself. Do you have any file or link about this elementary circuits? I have only my high school textbook and it covers only Ohm’s law and Kirchhoff’s law, resistors and capacitors with no mentions about LDRs, thermistors and diodes.
        Thank you so much

  102. i am begging for some good help.
    1) i do edexcel currently have given c1 c2 m1, and i am looking to the do the pat, what other edexcel math modules do i need?
    c3 c4? and if so do i need the whole book or specific chapters please inform me :((
    2) for the physics part do i need the entire of edexcel a2 physics?
    3) i do three subjects chem math and physics if i do further maths it can only be AS maths as my school does not even offer as further maths, would i need to show a predicted grade for this as i do not have the grade for AS further maths because i didnt do any of the modules or do i need it at all?

    1. HI — thanks for your question. In general, you should look at the link to the PAT syllabus above and check that you are covering the topics. Having had a brief look I think the summary is:
      1) I think that you will need a fair bit of the c3 and c4 modules, but there are parts that are not in the PAT syllabus.
      2) No you don’t need to cover all the a2 syllabus. The PAT physics syllabus is quite focused, so should be fairly easy to cover. The more mechanics you have done. the better.
      3) You don’t need to offer further maths A2. In any case, predicted grades aren’t that important, because admission is basically decided by PAT score and then interview performance. If you get an offer it will probably be AAA* or AAA, so you need to expect to get that level of grade.

      1. Thank you for your reply!
        In specifics could you tell what of A2 physics and c3,c4 to cover as time is valuable and I don’t want to waste it on things that aren’t needed
        Thank you for putting me at ease about further maths, but would it hinder my application in anyway even thoug Oxford require only 3 alevels,
        Lastly, I just saw the 2015 paper here and the solutions for the inequality question I didn’t understand what you did and I figured out it was from fp2 module edexcel so I thought myself that today, I’m thinking of going on the edexcel specification entering keywords from the syllabus and seeing where they appear are there any other modules in particular besides in c3 and c4 I should be aware of
        Sorry for nagging you but thank you for all the great work you do, I’m from dubai

        1. I think the best answer to your question is to look carefully at the syllabus, taking account of the way it has changed, using this link (https://oxfordpat.wordpress.com/changes-to-the-syllabus/) and just make sure you understand what each topic mentioned in the syllabus is getting at — searching the A level specifications is a good idea.

          If you read the Oxford physics entrance material on their web site, they say pretty clearly that it would make no difference to your application whether or not you study Further Maths, and they are trying to be as open and accurate as possible: they really mean everything that they say. So don’t worry about having to study Further Maths.

          The inequalities question requires you to know that if a > b then if you multiply both sides by the negative number x you get ax < bx. I don’t know much about the A level modules, but by googling around I think for at least some boards this is covered in C1 (see https://en.wikibooks.org/wiki/A-level_Mathematics/OCR/C1/Error_bounds_and_Inequalities#Solving_Inequalities_involving_Fractions).

          I hope this helps — to summarize, I am afraid I am no expert on the contents of the various A level modules, but I would recommend that you just look directly at the PAT syllabus and then follow up on those bits that look unfamiliar to you.

          1. Thank you again
            Did the question in the 2015 paper on Inequalities reslly only require that because I think they’re called rational inequalities and I wasn’t able to solve it or really understand your solution so I looked at tutorial on how to do them and I’m going to attempt the question again tomorrow

  103. I sat PAT last year. Based on my performance and other factors, I was interviewed at Oxford to study Engineering Science.
    Not long after the interview, I was offered a place at New College.
    Not long after that, I was offered a full Reach Oxford Scholarship that enables me to study at Oxford University from October 2016. My PAT performance was partly considered in this process.
    Of course, I have been working hard to earn those achievements. But I’m sure I would have faced many more struggles If you didn’t help me to develop my confidence in solving PAT problems.
    Thank you very much for your help. I couldn’t afford to have a tutor and your work substitutes private tuition in a much better way!!
    I hope you will continue with this work and I wish to support you by making a donation.
    Please can you advise ?

    1. Thank you so much for your comment: all the effort that I have put in to this site is made worthwhile by the knowledge that it has been useful.

      You ask about making donations. As it turns out, I am lucky enough to not need any financial support. Space on wordpress.com costs nothing, so the only thing I have ever spent on this site is my time, and we all have plenty of that. I don’t want to suggest any charity that you could donate to, because I don’t think my judgement would be any better than yours, and I know that life in Oxford is expensive, so you would be well-advised to hang on to your cash. Anyway, what goes around comes around: I was taught at one of the best schools in the UK for free; I always felt grateful to my teachers, and so years later I set up this web site so I could give people a little bit of the coaching that I had. You might well find yourself in a similar position one day, and for you to help somebody else in turn would be the best donation you could give.

  104. Wah…impressive web! Beautiful and useful! A thousand of thanks to you! It offer a lot of help to me!

    1. Hi Raya — do you mean UK year 10 or India class 10? If you mean UK, the answer is easy: just do A level Maths, Physics and ideally Further Maths. If you mean India, I’m afraid I’m not sure what you will already have studied, but you should look in detail at the syllabus on the Oxford PAT website. In a week or two I will be posting some information about how the syllabus has changed over the years and how this impacts the past papers.

  105. Hi Adam, thanks for your comment.

    Yes I would have been inclined to answer C to this because the question seems to be saying that there was some radiation emitted that was stopped by the aluminum plate, even though the emitted radiation was barely above background. The good news is that, now the syllabus has changed, this sort of question won’t come up.

    On the 9i answer, yes that is right. I think a better way of writing down this answer would be that die 1 could be a member of {1,2,3,4,5}, where each of these outcomes has probability 1/6, and for each of those outcomes there is a 1/6 probability of throwing a number on the second die such that the two numbers add up to 6. Therefore the total probability of throwing numbers that add up to six is 5/36.

    Many thanks for pointing this out. It is nice to be admitting to somebody else’s error for a change :-).

  106. Hi
    This site has been extremely helpful ! I had a doubt. Is showing our working important or is just the answer they are looking for? I’m unfamiliar with the marking scheme.

    1. Hi, I’m really glad the site has been useful. Yes, I think that showing your working is really important; it doesn’t matter how you show your working as long as the examiners can see that you have understood the maths and physics involved in the problem and can use your knowledge to solve it.

  107. Hi, I was hoping you could help me out with question 14 of the new specimen paper. For the second part, why is the velocity 3u rather than just u? I’d really appreciate any help. Thanks.

  108. Hi, do you know if there is a list of equations that we need to know? It’s just I don’t know all of the ones from the formula sheet by heart and I know which ones I should definitely learn but am unsure about several others such as decay of charge because we haven’t covered some of these topics at school and I have learnt the topic but don’t know as to how often they would come up in a paper?
    Thanks

    1. Hi,

      Can’t seem to add a comment on the solution page but I think the answer to Q10 on the 2015 Specimen paper is pi/3?? (Arccos(1/2)=pi/3)

      Thanks very much for the amazing website, it’s a great help

      Hi Ali — copied over to Question 10 (comments had got disabled somehow). Many thanks for correcting my careless slip.

    1. ok thank you- i know the new sample paper is almost the same as 2014 but these questions weren’t on the 2014 paper

  109. Hi Ben, thanks for your question.

    You mean the question: “Positron Emission Tomography (PET) scanners frequently operate using the radioactive isotope 18F, which has a half life of about two hours. The isotope is incorporated into a drug, half of which is excreted by the body every two hours. How long will it take before the quantity of radioactive drug in the body halves?”.

    Suppose the initial amount of radioactive drug is D. If the half life is T then the function for amount of radioactive drug left after time t is D 2^(-t/T) (i.e. ” D times two to the minus t over T”). If half the remaining drug is excreted every T seconds then our function becomes D 2^(-t/T) 2^(-t/T), which equals D 2^(-2t/T). So the resulting formula looks like the formula for radioactive decay when the half life is T/2. So, when T is 2 hours the effective half life is 1 hour, so the answer is B.

    1. Dear Sir/Madam,
      Thank you for your prompt response. I have understood the problem now. It is very nice of you to help us. And by the way, when you mention: ‘half the remaining drug is excreted every T seconds’, do you mean every hours since the unit of half-life were hour in the question?

  110. Hello, would you be able to have a look at question 7 (multiple choice) physics paper 2007 ? I haven’t been able to derive an equation to describe the rate of change when 2 simultaneous change happening at the same time. I would be very greatful for any advice on how to tackle this question. Thank you very much.

Leave a reply to Oxford PAT Cancel reply